Chapter 5 : The second law of Thermodynamics

Example 5.1 page no : 71

In [2]:
# Variables
Tc = 295.;			#K
Th = 585.;			#K
W = 800000.;			#KW

# Calculations
n_max = 1-(Tc/Th);
n = round(0.7*n_max,3)
Qc = round(((1-n)/n)*W,-2);

# Results
print 'Heat required',Qc,'KW'
Heat required 1505500.0 KW

Example 5.3 page no : 72

In [3]:
import math 

def MCPS(T0,T,A,B,C,D):
    t = T/T0;
    return (A)+(((B*T0)+(((C*T0*T0)+(D/(t*t*T0*T0)))*(t+1)/2))*((t-1)/math.log(t)))


# Variables
P2 = 1.;			#bar
P1 = 5.;			#bar
T0 = 550.;			#K
A = 1.702;
B = 9.081*(10**-3);
C = -2.164*(10**-6);
D = 0;

#Equation to be used
#(<Cp>s/R)ln(T2/T1) = ln(P2/P1) math.since del_S = 0
#let I = (<Cp>s/R)

# Calculations
a = T0-1;			#Initial
i = -1;
while (i == -1):
    b = MCPS(T0,a,A,B,C,D);
    c = (math.log(1./5))/(math.log(a/T0));
    flag = c-b;
    if(flag <= 0.0001):
        T = a;
        i = 1;
    else:
        a = a-.01; 
        i = -1;

# Results
print 'Final Temperature',T,'K'
Final Temperature 411.33 K

Example 5.4 page no : 73

In [4]:
import math 
from scipy.integrate import quad 

# Variables

#For Casting
Cp_Casting = 0.5;			#[KJ/Kg/K]
T1 = 723.15 	    		#[K]
T0 = 298.15	    	    	#[K]
M_Casting = 40.			    #[Kg]
#For Oil
Cp_Oil = 2.5    			#[KJ/Kg/K]
M_Oil = 150.	    		#[Kg]

# Calculations and Results
T = ((T1*M_Casting*Cp_Casting)+(T0*M_Oil*Cp_Oil))/((M_Casting*Cp_Casting)+(M_Oil*Cp_Oil));

#(a)-change in entropy For casting
def f1(T): 
    return 1./T

del_S_Casting = round(M_Casting*Cp_Casting* quad(f1,T1,T)[0],2)

print '(a)Change In Entropy of Casting',del_S_Casting,'KJ/K'

#(b)-change in entropy For Oil
def f2(T): 
    return 1./T

del_S_Oil = round(M_Oil*Cp_Oil* quad(f2,T0,T)[0],2)

print '(b)Change In Entropy of Oil',del_S_Oil,'KJ/K'

#(c)-Total
del_S_total = del_S_Casting+del_S_Oil;
print '(c)Total entropy change',del_S_total,'KJ/K'
(a)Change In Entropy of Casting -16.33 KJ/K
(b)Change In Entropy of Oil 26.13 KJ/K
(c)Total entropy change 9.8 KJ/K

Example 5.5 page no : 75

In [6]:
import math 

# Variables

#Gas A
rn_A = 1.;			#rate[mol/s]
T_A = 600.;			#[K]

#Gas B
rn_B = 2.;			#rate[mol/s]
T_B = 450.;			#[K]

#product
rn = rn_A+rn_B;			#[mol/s]
T = 400.;			#[K]
R = 8.314;
Cp = (7./2)*R;
T_s = 300.;			#Temperature[K]

# Calculations
#By equation (2.30) rQ = rn*H-rn_A*H_A-rn_B*H_B = rn_A(H-H_A)+rn_B*(H-H_B)  Rate of heat transfer
rQ = (rn_A*Cp*(T-T_A))+(rn_B*Cp*(T-T_B));			#[J/s] or  [W]
#By eqn (5.22) rSg = rn*S-(rn_A*S_A)-(rn_B*S_B)-(rQ/T_s)  rate of entropy generation for the process
rSg = round((rn_A*Cp*math.log(T/T_A))+(rn_B*Cp*math.log(T/T_B))-(rQ/T_s),3);			#[J/K/s]

# Results
print 'Rate of heat transfer',rQ,'J/s or W'
print 'Rate of entropy generation',rSg,'J/K/s'
Rate of heat transfer -8729.7 J/s or W
Rate of entropy generation 10.446 J/K/s

Example 5.6 page no : 77

In [7]:
import math 

# Variables
#Saturated Steam
#At T = 373.15K 
H1 = 2676.	    		#[KJ/Kg] from Steam table(App F)
S1 = 7.3554		    	#[KJ/Kg/K] from steam table(App F)
#At T = 273.15K  Liquid water
H2 = 0;
S2 = 0;

T_sigma = 273.15;			#[K]
T_r = 473.15;		    	#[K]
Q_r = -2000.;			    #[KJ]

del_H = H2-H1;
Q = del_H;
Q_sigma = Q-Q_r;

# Calculations and Results
del_S = S2-S1;
#For Heat Reservoir at 473.15K
del_St_T_r = (-Q_r/T_r);			#[KJ/K]
#For Heat reservoir provided by cooling water at 273.15K
del_St_T_sigma = -Q_sigma/T_sigma;
del_S_total = del_S+del_St_T_r+del_St_T_sigma;
print ('Since del_S_total<0 Process not feasible')

#Actual
Q_r = round((T_r/(T_r-T_sigma))*(del_H-(T_sigma*del_S)),1);
print 'Actual Heat transfer',Q_r,'KJ/Kg'
Since del_S_total<0 Process not feasible
Actual Heat transfer -1577.7 KJ/Kg

Example 5.7 page no : 78

In [8]:
import math 

def ICPH(T0,T,A,B,C,D):
    t = T/T0;
    return (A+((B/2)*T0*(t+1))+((C/3)*T0*T0*((t**2)+t+1))+(D/(t*T0*T0)))*(T-T0)

def ICPS(T0,T,A,B,C,D):
    t = T/T0;
    return ((A)*math.log(t))+(((B*T0)+(((C*T0*T0)+(D/(t*t*T0*T0)))*(t+1)/2))*(t-1))


# Variables
P1 = 50.			#bar
P2 = 1.013			#bar
T1 = 800.			#[K]
T2 = 300.			#[K]
R = 8.314

# Calculations
#del_H = intergral(CpdT) in the limits T1 and T2
A = 3.280;
B = 0.593*(10**-3);
C = 0;
D = 0.040*(10**5);
del_H = R*ICPH(T1,T2,A,B,C,D);			#[J/mol]

#del_S = integral[Cp(dT/T)] -Rln(P2/P1)  btw the limits T1,T2
del_S = (R*ICPS(T1,T2,A,B,C,D))-(R*math.log(P2/P1));			#[J/mol/K]
W_ideal = round(del_H-(T2*del_S),0);			#[J/mol]

# Results
print 'Maximum Work',W_ideal,'J/mol'
Maximum Work -15974.0 J/mol

Example 5.8 page no : 81

In [9]:
import math 

# Variables
#Saturated Steam
#At T = 373.15K 
H1 = 2676.;			#[KJ/Kg] from Steam table(App F)
S1 = 7.3554;			#[KJ/Kg/K] from steam table(App F)
#At T = 273.15K  Liquid water
H2 = 0;
S2 = 0;

T_sigma = 273.15;			#[K]
T_r = 473.15;			#[K]

# Calculations
del_H = H2-H1;
del_S = S2-S1;
W_ideal = del_H-(T_sigma*del_S);			#[KJ/Kg]
Q = round(abs(W_ideal*(T_r/(T_sigma-T_r))),1);			#[KJ]

# Results
print 'Maximum Possible Work',Q,'KJ'
Maximum Possible Work 1577.7 KJ

Example 5.9 page no : 83

In [12]:
%matplotlib inline

import math 
from matplotlib.pyplot import plot,suptitle,xlabel,ylabel


# Variables
T_H1 = 400.;			#[K]
T_H2 = 350.;			#[K]
T_C1 = 300.;			#[K]
T_sigma = 300.;			#[K]
rn_H = 1.;			#[mol/s]
R = 8.314;
Cp = (7./2)*R;

T_C2_a = T_H2-10;
T_C2_b = T_H1-10;

# Calculations and Results

X = [0,1];
Y = [T_C1,T_C2_a];
plot(X,Y);
Y = [T_H1,T_H2];
plot(X,Y);

X = [1,1];
Y = [290,410];
plot(X,Y);
X = [0,0.25];
Y = [T_C1,T_C1];
plot(X,Y,'--');
Y = [T_H1,T_H1];
plot(X,Y,'--');
X = [0.75,1];
Y = [T_C2_a,T_C2_a];
plot(X,Y,'--');
Y = [T_H2,T_H2];
plot(X,Y,'--');
suptitle("(a)Case 1,Cocurrent")
xlabel("Qc")
ylabel("T");

X = [0,1];
Y = [T_C1,T_C2_b];
plot(X,Y);
Y = [T_H2,T_H1];
plot(X,Y);
X = [1,1];
Y = [290,410];
plot(X,Y);
X = [0,0.25];
Y = [T_C1,T_C1];
plot(X,Y,'--');
Y = [T_H2,T_H2];
plot(X,Y,'--');
X = [0.75,1];
Y = [T_C2_b,T_C2_b];
plot(X,Y,'--');
Y = [T_H1,T_H1];
plot(X,Y,'--');
suptitle("(b)Case 2,Countercurrent")
xlabel("Qc")
ylabel("T");

#Solution
#Equation to be used
#(rn_H*Cp(T_H2-T_H1))+(rn_C*Cp(T_C2-T_C1)) = 0  Eq(A)
#del_rS = rn_H*Cp*(ln(T_H2/T_H1)+kln(T_C2/T_C1))  k = rn_C/rn_H  r-->Rate  Eqn(B)
#rW_lost = T_sigma*del_rS  Eqn(C)

#(a)-Cocurrent
#by Eqn(A)
T_C2_a = T_H2-10;
k = (T_H1-T_H2)/(T_C2_a-T_C1);			#k = rn_C/rn_H
#By Eqn(B)
del_rS = round(rn_H*Cp*(math.log(T_H2/T_H1)+(k*math.log(T_C2_a/T_C1))),3);			#[J/K/s]
#By Eqn(C)
rW_lost = round(T_sigma*del_rS,1);			#[J/s]or[W]
print ('(a)-Cocurrent')
print 'Rate Of change of entropy',del_rS,'J/K/s'
print 'Lost Work',rW_lost,'J/s or W'

#(b)-Countercurrent
T_C2_b = T_H1-10;
k = (T_H1-T_H2)/(T_C2_b-T_C1);			#k = rn_C/rn_H
#By Eqn(B)
del_rS = round(rn_H*Cp*(math.log(T_H2/T_H1)+(k*math.log(T_C2_b/T_C1))),3);			#[J/K/s]
#By Eqn(C)
rW_lost = round(T_sigma*del_rS,1);			#[J/s]or[W]
print ('(b)-Countercurrent')
print 'Rate Of change of entropy',del_rS,'J/K/s'
print 'Lost Work',rW_lost,'J/s or W'
(a)-Cocurrent
Rate Of change of entropy 0.667 J/K/s
Lost Work 200.1 J/s or W
(b)-Countercurrent
Rate Of change of entropy 0.356 J/K/s
Lost Work 106.8 J/s or W